Calculate the Variance of a Linear Combination

Click For Summary
To calculate the variance of a linear combination of variables (b1, b2, b3, b4, b5, b6), the correct formula involves the variances of each variable and their covariances. The initial attempt incorrectly squared the variances and misrepresented the covariance terms. A user pointed out the need for clarity in notation, suggesting that the expression should reflect the sum of the variables rather than a list. The discussion emphasizes the importance of accurately applying the variance formula to avoid errors. Understanding the correct formulation is crucial for proper statistical analysis.
tnqz44
Messages
1
Reaction score
0

Homework Statement



I want to calculate the variance of a linear combination (b1, b2, b3, b4, b5, b6). I know what the variance equation is but I'm not sure if I have expanded it right.

Homework Equations



http://en.wikipedia.org/wiki/Variance

The Attempt at a Solution



Var(b1, b2, b3, b4, b5, b6) = Var(b1)^2 + Var(b2)^2 + Var(b3)^2 + Var(b4)^2 + Var(b5)^2 + Var(b6)^2 + 2Cov(b1,b2) + 2Cov(b1,b3) + 2Cov(b1,b4)+ 2Cov(b1,b5) + 2Cov(b1,b6) + 2Cov(b2,b3)+ 2Cov(b2,b4) + 2Cov(b2,b5) + 2Cov(b2,b6) + 2Cov(b3,b4)+ 2Cov(b3,b5) + 2Cov(b3,b6) + 2Cov(b4,b5) + 2Cov(b4,b6) + 2Cov(b5,b6)
Please tell me if I got it correct, very much appreciated :)
 
Physics news on Phys.org
tnqz44 said:

Homework Statement



I want to calculate the variance of a linear combination (b1, b2, b3, b4, b5, b6). I know what the variance equation is but I'm not sure if I have expanded it right.

Homework Equations



http://en.wikipedia.org/wiki/Variance

The Attempt at a Solution



Var(b1, b2, b3, b4, b5, b6) = Var(b1)^2 + Var(b2)^2 + Var(b3)^2 + Var(b4)^2 + Var(b5)^2 + Var(b6)^2 + 2Cov(b1,b2) + 2Cov(b1,b3) + 2Cov(b1,b4)+ 2Cov(b1,b5) + 2Cov(b1,b6) + 2Cov(b2,b3)+ 2Cov(b2,b4) + 2Cov(b2,b5) + 2Cov(b2,b6) + 2Cov(b3,b4)+ 2Cov(b3,b5) + 2Cov(b3,b6) + 2Cov(b4,b5) + 2Cov(b4,b6) + 2Cov(b5,b6)


Please tell me if I got it correct, very much appreciated :)

By Var(b1,b2,b3,b4,b5) do you mean Var(b1+b2+b3+b4+b5)? If so, why not write it properly?

Anyway, if you do mean Var(b1+b2+b3+b4+b5), then your formula is WRONG. I don't want to say more, because that would be giving the solution, but I will just say: go back and read what the formula in your link says, then look *very carefully* at what you have written. Can you see the difference?

RGV
 
Question: A clock's minute hand has length 4 and its hour hand has length 3. What is the distance between the tips at the moment when it is increasing most rapidly?(Putnam Exam Question) Answer: Making assumption that both the hands moves at constant angular velocities, the answer is ## \sqrt{7} .## But don't you think this assumption is somewhat doubtful and wrong?

Similar threads

Replies
1
Views
2K
  • · Replies 8 ·
Replies
8
Views
2K
Replies
235
Views
14K
  • · Replies 7 ·
Replies
7
Views
2K
  • · Replies 2 ·
Replies
2
Views
2K
  • · Replies 1 ·
Replies
1
Views
1K
  • · Replies 1 ·
Replies
1
Views
2K
  • · Replies 4 ·
Replies
4
Views
2K
  • · Replies 6 ·
Replies
6
Views
10K
  • · Replies 17 ·
Replies
17
Views
2K